X^y = y^x

Discussion générale entre passionnés et amateurs de mathématiques sur des sujets mathématiques variés
Mikou
Membre Rationnel
Messages: 910
Enregistré le: 06 Nov 2005, 14:17

par Mikou » 24 Mar 2006, 19:32

ironie cher yos ?
la pb est donc pour quel valeures x,y reeles distinctes a ton legalite x^y = y^x verifiée tel que x^y soit maximum : reponse il ny en a pas !



montsegur
Membre Relatif
Messages: 144
Enregistré le: 19 Mar 2006, 13:04

par montsegur » 24 Mar 2006, 19:51

il y a un minimum de x^y = y^x lorsque x = y = e
Dès que x est différent de y avec :
x compris entre 1 et e et y compris entre e et l'infini
ou l'inverse en permutant x et y
x^y = y^x est supérieur à e^e
C'est une constataion numérique.
Le problème est de le démontrer.

Mikou
Membre Rationnel
Messages: 910
Enregistré le: 06 Nov 2005, 14:17

par Mikou » 24 Mar 2006, 19:54

ce n'est pas le minimum si on prends x=y=1. par contre pour x et y distincts on peut montrer 'assez facilement' qu 'il ny a pas de max.

montsegur
Membre Relatif
Messages: 144
Enregistré le: 19 Mar 2006, 13:04

par montsegur » 24 Mar 2006, 21:03

J'ai commencé une démonstration, mais je n'abouti pas.
Si cela intéresse quelqu'un, je peux la poster.
A défaut d'autre chose, cela peut donner une piste,
mais je crains que ce soit une impasse.

yos
Membre Transcendant
Messages: 4858
Enregistré le: 10 Nov 2005, 21:20

par yos » 24 Mar 2006, 21:25

Mikou a écrit:ironie cher yos ?


Pas le moins du monde pour une fois.

la pb est donc pour quel valeures x,y reeles distinctes a ton legalite x^y = y^x verifiée tel que x^y soit maximum : reponse il ny en a pas !


Non je ne parle pas de maximum, simplement de l'ensemble des couples (x,y) de réels positifs vérifiant x^y=y^x. Ca fait bien une courbe. A quoi elle ressemble?

montsegur
Membre Relatif
Messages: 144
Enregistré le: 19 Mar 2006, 13:04

par montsegur » 25 Mar 2006, 07:52

Je propose une piste :

z = x^y = y^x ; donc : ln(z) = y ln(x) = x ln(y) = Z

Donc : [ln(x)]/x = [ln(y)]/y = a

D'où : ln(x) = a x et ln(y) = a y

Donc : ln(z) = y ln(x) = axy = x ln(y)

D'où : z = e^(axy)

Z' = y/x + ln(x) = x/y + ln(y) = 0

entraîne alors : ln(x) = -y/x et ln(y) = -x/y = 1/ln(x)

Donc : ay = ln(y) = 1/ln(x) = 1/ax d'où : y = 1/ (a²x)

D'où : z = e^(axy) = e^(ax/a²x) = e^(1/a)

Posons : y1 = ln(x) et y2 = a x ; alors : y'1 = 1/x et y'2 = a

Donc pour y'1 = y'2 on obtient : 1/x = a et donc : x = 1/a

D'où : ln(x) = ax entraîne ln(1/a) = a (1/a) = 1 donc 1/a = e

et a = 1/e ce qui entraîne que z = e^e

Au point x = e ; y1 = ln(x) = ln(e) = 1 et y2 = ax = (1/e) e = 1

Sur un graphique, on voit bien que y2 = ax ne coupe y1 = ln(x)

que pour a inférieur à 1/e

Autre graphique : y1 = a et y2 = x^(1/x)

ln(y2) = (1/x) ln(x) = z d'où : z' = (1/x²) [ ln(x) - 1 ]

z' = o entraîne x = e

y2 passe par un maximum = e^(1/e) pour x = e

Maintenant, y1 = a ne coupe y2 que pour a inférieur à e^(1/e)

Mais y2 a une asymptote en y2 = 1

Donc pour y2 compris entre 1 et e^(1/e)

l'axe des x est partagé en deux régions :

A^B = B^A pour lesquels :

A est compris entre 1 et e

B est compris entre e et l'infini

montsegur
Membre Relatif
Messages: 144
Enregistré le: 19 Mar 2006, 13:04

par montsegur » 25 Mar 2006, 09:11

Dans le post précédant, j'ai fait les calculs en considérant

que x et y étaient indépendants ce qui est faux car x et y sont liés.

Je reprends donc les calculs avec y = f(x) :

z = x^y = y^x ; donc : ln(z) = y ln(x) = x ln(y) = Z

Z' = y' ln(x) + y/x = (x/y) y' + ln(y) = 0 entraîne :

y' = -(y/x) ln(y) = -(y/x) [ 1/ln(x) ] d'où : ln(y) = 1/ln(x)

Cette égalité n'est satisfaite que pour : x = y = e et x = y = 1/e

Donc Z = ln(z) est extrémal pour : x = y = e et x = y = 1/e

Mais Z = ln(z) donc Z' = z'/z et alors : z' = z Z' soit :

z' = z [ y' ln(x) + y/x ] = z [ x/y) y' + ln(y) ]

Comme z n'est jamais nul, les extréma de Z sont aussi les extréma de z

x^y = y^x implique que x et y sont liés et donc que y = f(x)

Le calcul de z' = 0 par Z' = 0 nous donne ln(y) = 1/ln(x)

et donc nous en déduisons que y = f(x) = e^(1/ln(x))

x = y = 1/e donne Z = - e et z = (1/e)^(1/e)

x = y = e donne Z = + e et z = e^e

Graphiquement : x = y = e est valide,

alors que x = y = 1/e ne convient pas

Il faut donc justifier que seul : x = y = e est valide

pour que x^y = y^x soit minimum.

yos
Membre Transcendant
Messages: 4858
Enregistré le: 10 Nov 2005, 21:20

par yos » 25 Mar 2006, 11:29

montsegur a écrit:Il faut donc justifier que seul : x = y = e est valide

pour que x^y = y^x soit minimum.


...
Bizarre ton minimum.

montsegur
Membre Relatif
Messages: 144
Enregistré le: 19 Mar 2006, 13:04

par montsegur » 25 Mar 2006, 11:34

Yos dit : Bizarre ton minimum.

Ta réponse ne convient pas

car x et y ne sont pas indépendants

mais liés par x^y = y^x

yos
Membre Transcendant
Messages: 4858
Enregistré le: 10 Nov 2005, 21:20

par yos » 25 Mar 2006, 12:35

montsegur a écrit:Yos dit : Bizarre ton minimum.

Ta réponse ne convient pas

car x et y ne sont pas indépendants

mais liés par x^y = y^x


Selon toi 2^2 n'est pas égal à 2^2 ??

montsegur
Membre Relatif
Messages: 144
Enregistré le: 19 Mar 2006, 13:04

par montsegur » 25 Mar 2006, 12:46

Hé Yos...

Agites les boyaux de la tête, les neurones, les synapses...

Tu ne vois clair dans ce problème.

yos
Membre Transcendant
Messages: 4858
Enregistré le: 10 Nov 2005, 21:20

par yos » 25 Mar 2006, 14:02

montsegur a écrit:Hé Yos...

Agites les boyaux de la tête, les neurones, les synapses...

Tu ne vois clair dans ce problème.


Ca doit être ça.

montsegur
Membre Relatif
Messages: 144
Enregistré le: 19 Mar 2006, 13:04

par montsegur » 25 Mar 2006, 16:48

J'ai modifié et amélioré le post n° 24

yos
Membre Transcendant
Messages: 4858
Enregistré le: 10 Nov 2005, 21:20

par yos » 25 Mar 2006, 17:10

d'où : ln(y) = 1/ln(x)

Cette égalité n'est satisfaite que pour : x = y = e et x = y = 1/e


T'es sûr de ça?

On trouve y= exp(1/lnx).

Je conviens que e et 1/e sont solutions mais le fait que ce soient les seules est pas immédiat. Cela dit ton minimum je le répète est peut-être bien obtenu en 1/e mais sûrement pas en e.

montsegur
Membre Relatif
Messages: 144
Enregistré le: 19 Mar 2006, 13:04

par montsegur » 25 Mar 2006, 18:24

Yos dit : T'es sûr de ça?

Réponse :

y = f(x) = e^(1/ln(x)) est la conséquence

du calcul des extréma de : x^y = y^x

Bien sûr que : ln(y) = 1/ln(x) est satisfait

par d'autres valeurs que e et 1/e

J'ai donc eu tord de dire que :

Cette égalité n'est satisfaite que pour : x = y = e et x = y = 1/e

Mais une étude graphique m'a orienté vers ces valeurs

car elles sont des cas particuliers simples.

Les tracés de y1 = ln(x) et de y2 = a x ( avec a = 1/e voir post 23 )

nous montrent que seul e convient. Mais c'est une constattion

graphique, pas une démonstration mathématique.

Donc Yos, d'accord pour ta remarque. Nous devons encore chercher.

montsegur
Membre Relatif
Messages: 144
Enregistré le: 19 Mar 2006, 13:04

par montsegur » 25 Mar 2006, 22:41

Voilà enfin la démonstration :

z = x^y = y^x ; donc : ln(z) = y ln(x) = x ln(y) = Z

Z' = y' ln(x) + y/x = (x/y) y' + ln(y) = 0 entraîne :

y' = -(y/x) ln(y) = -(y/x) [ 1/ln(x) ] d'où : ln(y) = 1/ln(x)

et alors : y = f(x) = e^[1/ln(x)] que nous reportons dans z = y^x

Donc : z = { e^[1/ln(x)] }^x = e^[x/ln(x)] d'où : ln(z) = x/ln(x)

z'/z = 1/ln(x) + ( x ) [ -1/(ln(x))² ] ( 1/x ) = [ 1/ln(x) ] [ 1-1/ln(x) ]

z' = [ z/ln(x) ] [ 1-1/ln(x) ] = 0 entraîne : 1 = 1/ln(x) et donc x = e

Donc : ln(y) = 1/ln(x) = 1/ln(e) = 1 cela donne y = e

Voilà enfin la démonstration que x = y = e

Mais maintenant, il faut prouver que nous avons un minimum.

Pour cela, il nous faut calculer z"

si z" est positif pour x = e alors z est minimum

si z" est négatif pour x = e alors z est maximum

z' = [ z/ln(x) ] [ 1-1/ln(x) ] = z [ 1/ln(x) - 1/ln²(x) ] = z v

z" = v z' + z v' mais pour x = e on a z' = 0 donc z" = z v'

v = 1/ln(x) - 1/ln²(x) donc v' = -1/[x (ln(x)²)] + 2/[x (ln(x))^3]

et v' = [ 1/(x (ln(x)²) ] [ -1 + 2/ln(x) ] = 1/e pour x = e

Donc : z" = z v' = z/e mais z = e^[x/ln(x)] = e^e pour x = e

Finalement z" = (e^e)/e qui est positif. Donc z = x^y = y^x

est bien minimum pour x = y = e CQFD

montsegur
Membre Relatif
Messages: 144
Enregistré le: 19 Mar 2006, 13:04

par montsegur » 26 Mar 2006, 11:09

Il est possible que ma démonstration soit fausse.
Critiques et corrections sont les bienvenues.

montsegur
Membre Relatif
Messages: 144
Enregistré le: 19 Mar 2006, 13:04

par montsegur » 27 Mar 2006, 11:25

Je viens de vérifier sur la calculatrice TI-92 que :

z = x^y = y^x = { e^[1/ln(x)] }^x

passe bien par le minimum e^e pour x = e

Donc la démonstration du post précédent est exacte.

L'affaire est donc classée.

montsegur
Membre Relatif
Messages: 144
Enregistré le: 19 Mar 2006, 13:04

par montsegur » 27 Mar 2006, 19:16

J'ai vérifié sur une calculatrice graphique que :

z = x^y = y^x = { e^[1/ln(x)] }^x

passe bien par le minimum e^e pour x = e

Donc la démonstration sur le post précédent est exacte.

L'affaire est donc classée.

montsegur
Membre Relatif
Messages: 144
Enregistré le: 19 Mar 2006, 13:04

par montsegur » 28 Mar 2006, 10:57

Quidam ne poste plus. Que se passe-t-il ?

Retourner vers ⚜ Salon Mathématique

Qui est en ligne

Utilisateurs parcourant ce forum : Aucun utilisateur enregistré et 4 invités

Tu pars déja ?



Fais toi aider gratuitement sur Maths-forum !

Créé un compte en 1 minute et pose ta question dans le forum ;-)
Inscription gratuite

Identification

Pas encore inscrit ?

Ou identifiez-vous :

Inscription gratuite